Difference between revisions of "2002 AMC 8 Problems/Problem 13"

(Created page with "For his birthday, Bert gets a box that holds 125 jellybeans when filled to capacity. A few weeks later, Carrie gets a larger box full of jellybeans. Her box is twice as high, twi...")
 
(Solution)
 
(9 intermediate revisions by 6 users not shown)
Line 1: Line 1:
 +
==Problem==
 +
 
For his birthday, Bert gets a box that holds 125 jellybeans when filled to capacity. A few weeks later, Carrie gets a larger box full of jellybeans. Her box is twice as high, twice as wide and twice as long as Bert's. Approximately, how many jellybeans did Carrie get?  
 
For his birthday, Bert gets a box that holds 125 jellybeans when filled to capacity. A few weeks later, Carrie gets a larger box full of jellybeans. Her box is twice as high, twice as wide and twice as long as Bert's. Approximately, how many jellybeans did Carrie get?  
  
 
<math> \text{(A)}\ 250\qquad\text{(B)}\ 500\qquad\text{(C)}\ 625\qquad\text{(D)}\ 750\qquad\text{(E)}\ 1000 </math>
 
<math> \text{(A)}\ 250\qquad\text{(B)}\ 500\qquad\text{(C)}\ 625\qquad\text{(D)}\ 750\qquad\text{(E)}\ 1000 </math>
 +
 +
==Solution==
 +
Since the volume ratio is equal to the sides ratio cubed, then the ratio of the larger box's volume to the smaller one is 2 cubed.
 +
 +
<math>2^3=8</math>
 +
 +
Now multiply 125 (the number of jellybeans that Bert's box can hold) by 8.
 +
 +
<math>8\cdot125=
 +
\boxed{\text{(E)}\ 1000}</math>.
 +
 +
==See Also==
 +
{{AMC8 box|year=2002|num-b=12|num-a=14}}
 +
{{MAA Notice}}

Latest revision as of 10:07, 18 September 2021

Problem

For his birthday, Bert gets a box that holds 125 jellybeans when filled to capacity. A few weeks later, Carrie gets a larger box full of jellybeans. Her box is twice as high, twice as wide and twice as long as Bert's. Approximately, how many jellybeans did Carrie get?

$\text{(A)}\ 250\qquad\text{(B)}\ 500\qquad\text{(C)}\ 625\qquad\text{(D)}\ 750\qquad\text{(E)}\ 1000$

Solution

Since the volume ratio is equal to the sides ratio cubed, then the ratio of the larger box's volume to the smaller one is 2 cubed.

$2^3=8$

Now multiply 125 (the number of jellybeans that Bert's box can hold) by 8.

$8\cdot125= \boxed{\text{(E)}\ 1000}$.

See Also

2002 AMC 8 (ProblemsAnswer KeyResources)
Preceded by
Problem 12
Followed by
Problem 14
1 2 3 4 5 6 7 8 9 10 11 12 13 14 15 16 17 18 19 20 21 22 23 24 25
All AJHSME/AMC 8 Problems and Solutions

The problems on this page are copyrighted by the Mathematical Association of America's American Mathematics Competitions. AMC logo.png